Difference between revisions of "2022 AMC 10A Problems/Problem 2"

m (Problem)
Line 2: Line 2:
 
Mike cycled <math>15</math> laps in <math>57</math> minutes. Assume he cycled at a constant speed throughout. Approximately how many laps did he complete in the first <math>27</math> minutes?
 
Mike cycled <math>15</math> laps in <math>57</math> minutes. Assume he cycled at a constant speed throughout. Approximately how many laps did he complete in the first <math>27</math> minutes?
  
<math>\textbf{(A)} 5 \qquad\textbf{(B)} 7 \qquad\textbf{(C)} 9 \qquad\textbf{(D)} 11 \qquad\textbf{(E)} 13</math>
+
<math>\textbf{(A) } 5 \qquad\textbf{(B) } 7 \qquad\textbf{(C) } 9 \qquad\textbf{(D) } 11 \qquad\textbf{(E) } 13</math>
  
 
== Solution 1==  
 
== Solution 1==  

Revision as of 21:35, 14 December 2023

Problem

Mike cycled $15$ laps in $57$ minutes. Assume he cycled at a constant speed throughout. Approximately how many laps did he complete in the first $27$ minutes?

$\textbf{(A) } 5 \qquad\textbf{(B) } 7 \qquad\textbf{(C) } 9 \qquad\textbf{(D) } 11 \qquad\textbf{(E) } 13$

Solution 1

Mike's speed is $\frac{15}{57}=\frac{5}{19}$ laps per minute.

In the first $27$ minutes, he completed approximately $\frac{5}{19}\cdot27\approx\frac{1}{4}\cdot28=\boxed{\textbf{(B) } 7}$ laps.

~MRENTHUSIASM

Solution 2

Mike runs $1$ lap in $\frac{57}{15}=\frac{19}{5}$ minutes. So, in $27$ minutes, Mike ran about $\frac{27}{\frac{19}{5}} \approx \boxed{\textbf{(B) }7}$ laps.

~MrThinker

Solution 3

Mike's rate is \[\frac{15}{57}=\frac{x}{27},\] where $x$ is the number of laps he can complete in $27$ minutes. If you cross multiply, $57x = 405$.

So, $x = \frac{405}{57} \approx \boxed{\textbf{(B) }7}$.

~Shiloh000

Solution 4 (Quick Estimate)

Note that $27$ minutes is a little bit less than half of $57$ minutes. Mike will therefore run a little bit less than $15/2=7.5$ laps, which is about $\boxed{\textbf{(B) }7}$.

~UltimateDL

Solution 5 (Approximation)

Note that $57$ minutes is almost equal to $1$ hour. Running $15$ laps in $1$ hour is running approximately $1$ lap every $4$ minutes. This means that in $27$ minutes, Mike will run approximately $\frac{27}{4}$ laps. This is very close to $\frac{28}{4} = \boxed{\textbf{(B) }7}$.

~TheGoldenRetriever

Video Solution 1 (Quick and Easy)

https://youtu.be/tu4rE1nqY9g

~Education, the Study of Everything

See Also

2022 AMC 10A (ProblemsAnswer KeyResources)
Preceded by
Problem 1
Followed by
Problem 3
1 2 3 4 5 6 7 8 9 10 11 12 13 14 15 16 17 18 19 20 21 22 23 24 25
All AMC 10 Problems and Solutions

The problems on this page are copyrighted by the Mathematical Association of America's American Mathematics Competitions. AMC logo.png